Wednesday, May 11, 2011

100 MCQs of Socio Economic Development

1. L&T Infotech was chosen to be an Implementation Agency to implement the Business Analytics Project in order to provide for data and information for analysing insurance companies and regulatory decision making by which body?
a.    IRDA
b.    SEBI
c.    RBI
d.    PFO
Answer: (a)

2. Propelled by higher crude petroleum and steel production, the index of the six ‘core' infrastructure industries registered 6.8 per cent higher growth in March 2011 compared to March 2011. What percentage growth did the index record?
a.    7.4%
b.    7.1%
c.    7.5%
d.    7.34%
Answer: (a)

3. The Annual Monetary Policy for 2011-12 was presented by RBI Governor D.Subbarao on 3 May 2011. With regard to the monetary policies mention which of the following is/are not correct?
1.    RBI increased the repo rate under the liquidity adjustment facility (LAF) by 50 basis points from 6.75 per cent to 7.25 per cent with immediate effect.
2.    The Marginal Standing Facility (MSF) rate, determined with a spread of 100 basis points above the repo rate, stands calibrated at 8.75 per cent.
3.    The Bank Rate has been retained at 5.0 per cent.
4.    Reserve Bank of India (RBI) in May 2011 decided to accept the broad framework of regulations recommended by the Malegam Committee report on micro finance institutions (MFIs)
a.    3 & 4
b.    1 & 4
c.    2 & 3
d.    1 & 3
Answer: (c)

4. The Reserve Bank of India (RBI) capped bank investments into liquid schemes to what per cent of the bank’s net worth?
a.    15%
b.    10%
c.    12%
d.    13.2%
Answer: (b)

5. Which Industry Body proposed setting up of National Knowledge Functional Hub (NKFH) to engage higher educational institutions with the industry in order to produce quality engineering graduates and meet increasing requirement of skilled hands in the market?
a.    Food Corporation of India
b.    Federation of Indian Chambers of Commerce and Industry
c.    Biotechnology Council of India
d.    Confederation of Indian Industries
e.    ASSOCHAM
Answer: (b)

6. Which body in India demanded on 24 April 2011 introduction of financial support through legislation for preventing corrosion of machines, pipelines and industrial structures?
a. Confederation of Indian Industry
b. Indian Ministry of Commerce
c. Competition Commission of India
d. Indian Chamber of Commerce
Answer: (a)

7. Which of the following bodies in April 2011modified the norms for appointment of its internal auditor?
a. SEBI
b. PFRDA
c. IRDA
d. India Inc
Answer: (a)

8. According to the data released by Central Electricity Authority, India generated 811 billion units of electricity in 2010-11 fiscal marginally lower than the set target. What was the target set for electricity generation for 2010-11?
a. 830 billion units
b. 830.8 billion units
c. 815 billion units
d. 818 billion units
Answer: (b)

9. Market regulator SEBI on 25 April 2011 sought more time from the Supreme Court to give its views on the report of a high-powered committee that probed case that dates back to 2006. Name the scam being referred to.
a. IPO scam
b. Telgi Stamp Paper Scam
c. UTI Scam
d. Madhu Coda Scam
Answer: (a)

10. National Association of Software and Services Companies (NASSCOM) on 26 April 2011 announced which of the following individual’s appointment as its Chairman?
a. Rajendra S. Pawar
b. Harsh Manglik
c. N. Chandrasekaran
d. Ganesh Natarajan
Answer: (a)

11. The Reserve Bank of India on 26 April 2011 fined 19 banks, including the country's top private and foreign banks for violating its guidelines on derivatives. Which is the only public sector bank on the list of banks to be fined?
a. State bank of India
b. Punjab National Bank
c. Bank of India
d. Indian Bank
Answer: (a)

12. The Reserve Bank of India on 18 April 2011 formed a conglomerate cell within its supervisory set-up to keep a constant vigil on 12 large domestic and foreign banks. Which one of these banks will not come under the prescribed supervisory vigil?
a.State Bank of India
b. Bank of India
c. Bank of Baroda
d. Canara Bank
e. Punjab National Bank
Answer: (d)

13. Power equipment maker Bharat Heavy Electricals (BHEL) on 19 April 2011 announced that its consortium with Alstom won Rs 1,600 crore contract for steam turbine generators for which nuclear power station in India?
a. Kakrapur Nuclear Power Station
b. Kalpakkam Nuclear Power Station
c. Kudankulam Nuclear Power Plant
d. Jaitapur Nuclear Power Project
Answer: (a)

14. Oil regulator Directorate General of Hydrocarbons (DGH) refused to which companies’ spending on KG-D6 gas fields?
a. Reliance Industries
b. ONGC Videsh
c. Essar Oil
d. Mangalore Refinery and Petrochemicals Limited
Answer: (a)

15. Which of the following PSUs was honoured with the Outstanding PSU of the Year award by All India Management Association on 21 April 2011?
a. GAIL India
b. ONGC
c. Bharat Heavy Electricals Ltd.
d. Indian Oil Corporation Ltd.
Answer: (a)

16. Which public sector bank decided to do away with the teaser home loan scheme beginning 30 April 2011?
a. State Bank of India
b. Bank of India
c. Union Bank
d. Punjab National Bank
Answer: (a)

17. Read the sentences with respect to the Telecom Policy 2011.
1. Telecom Policy 2011 was unveiled by the Communications and IT Minister, Kapil Sibal on 11 April 2011.
2. The decision to move to a uniform licence fee in the telecom sector was also pronounced in the policy.
3. The norms concerning merger and acquisition (M&A) mentioned that the minimum number of operators will not be allowed to fall below eight at any point in time in each circle.
4. The policy highlighted formation of a committee, under the chairmanship of retired Supreme Court judge Shivraj Patil to draft the Telecom Act.
Which of the above mentioned facts are not true about Telecom Policy 2011?
a. 1,2 & 3
b. 3 & 4
c. 1 & 2
d. 2 & 3
Answer: (b)

18.  Indian Finance Ministry on 11 April decided to issue biometric PAN cards to taxpayers across the country to erase the problem of duplicate and fake ones. Which Indian Finance Minister has first proposed the biometric PAN card?
a. Pranab Mukherjee
b. P Chidambaram
c. Jaswant Singh
d. Yashwant Sinha
Answer: (b)

19. Which industry body representing GSM players announced in April 2011 its decision to combine forces with the government to ensure wide scale spread of mobile broadband?
a. Cellular Operators Association of India
b. Cellular Rights Association
c. TRAI’s Cellular Arm
d. DIT
Answer: (a)

20. The 218-km Chainsa-Sultanpur-Neemrana pipeline was commissioned on 9 April 2011. Which of the following built the pipeline?
a. RIGZONE
b. GAIL (India)
c. High - Tech Group of Companies
d. Max Engineering
Answer: (b)

21. According to global research group Gartner, the Indian domestic BPO market is projected to grow by 23% and reach which of the following figures in the current financial year 2011?
a. $1.69 billion
b. $1.4 billion
c. $2.47 billion
d. $1.1 billion
Answer: (b)

22. Which are the two Gujarat-based cooperative sector lenders on which Reserve bank of India imposed a financial penalty of Rs 1 lakh each for violation of various rules, including anti-money laundering guidelines?
1. Shree MahalaxmiMercantile Co-operative Bank
2. Rander People's Co-operative Bank
3. Abad District Co Op Bank
4. Ahd Mercantile Co-Op Bank Ltd
a. 1 & 2
b. 2 & 3
c. 1 & 4
d. 3 & 4
Answer: (a)

23. In view of the Intelligence Bureau’s report, Department of Telecommunications in April 2011requested to advice the Telecom Service Providers not to launch proposed pushmail/powermail service of which of the following?
a. HTC
b. Blackberry
c. Nokia
d. Apple’s iphone
Answer: (c)

24.    Which of the following steel plants in India achieved record production of 5.71 million tonnes (mt) of hot metal (a 6.3 per cent growth over 2009-10), 5.33 mt of crude steel (4.3 per cent) and 4.57 mt of saleable steel (4.3 per cent)?
a.    Bhillai Steel Plant
b.    Rourkela Steel Plant
c.    Bokaro Steel Plant
d.    Salem Steel Plant
Answer: (a)

25.    The Department of Information Technology (DIT) on 5 April 2011 published a draft policy on which of the following to enable the government departments to provide services, including payment of utility bills and filing of tax with the use of a medium?
a.    Mobile governance
b.    E-governance
c.    Service Delivery Gateway
d.    Applications Inter-operability
Answer: (a)

26.    Engineering and construction major L&T announced that it has achieved financial closure for which of the following with the help of 10-bank consortium led by SBI pitching in with Rs 11478 crore debt?
a.    Kolkata Metro Rail Project
b.    Pune Metro Project
c.    Delhi Metro project
d.    Hyderabad Metro Rail project
Answer: (d)

27.    At the National Conference on Kharif Strategies, Agriculture Minister Sharad Pawar on 6 April 2011 mentioned that India has achieved an all-time high production of foodgrains. What was the estimation regarding foodgain production in 2010-11?
a.    235.88 million tonnes
b.    215 million tonnes
c.    261.34 million tonnes
d.    222.66 million tonnes
Answer: (a)

28.    This agency issued guidelines on distance marketing of products on 6 April 2011 to protect the interest of people who buy policies over phone or Internet. Identify the agency.
a.    SEBI
b.    IRDA
c.    TRAI
d.    DoT
Answer: (b)

29.    Buyer's Credit was launched by the Commerce and Industry Minister, Anand Sharma, on 6 April 2011 to boost project exports from India. Under which of the following was Buyer’s Credit launched?
a.    National Export Insurance Account
b.    Export-Import Bank of India
c.    Export Credit Guarantee Corporation of India
d.    Reserve Bank of India
Answer: (a)

30.    Record food grain production helped to reduce to food inflation to a four month low for the week ending 26 March 2011. What was the inflation percentage for the week?
a.    9.18%
b.    9.25%
c.    9.21%
d.    9.36%
Answer: (a)

31. Read the following statements regarding FSLRC
1.    Indian government on 24 March 2011 constituted 11member Financial Sector Legislative Reforms Commission (FSLRC).
2.    The 11-member commission will be headed by former PFRDA Chairman D Swarup.
3.    The commission will examine the architecture of the legislative and regulatory system governing the financial sector in India.
4.    It will study the interplay of exchange controls under SEBI and IRDA Policy with other regulatory regimes within the financial sector.
Which of the above mentioned statements are not true?
a.    Only 4
b.    2 & 4
c.    3 & 4
d.    Only 3
Answer: (b)

32. The Indian government in end March 2011 announced its decision to create Rs 2,500-crore corpus for   technology modernisation of which of the following by 2012?
a.    Handloom sector
b.    Micro, small and medium units
c.    Horticulture sector
d.    Dairy
Answer: (b)

33.  National Thermal Power Corporation (NTPC Ltd) and Indian Ministry of Power signed Memorandum of Understanding (MOU) for the year 2011-12 on 27 March in New Delhi. What is the current total installed capacity of NTPC?
a.    33694 MW
b.    31684 MW
c.    35321 MW
d.    46099 MW
Answer: (a)

34.    The Cabinet Committee on Economic Affairs cleared two major foreign direct investment (FDI) proposals including that of Hero Investments Pvt. Ltd. (HIPL), a part of the Hero Group, and Reckitt Benckiser's plan. What is the total worth of the FDI proposals cleared by the government?
a.    Rs 7800 crore
b.    Rs 7850 crore
c.    Rs 8100 crore
d.    Rs 8800 crore
Answer: (a)

35.    Commodity exchange regulator Forward Markets Commission (FMC) in March end 2011 finally agreed to give six more months to which of the following bourses to resurrect its equity structure?
a.    MCX
b.    NCDEX
c.    NMCE
d.    NSE
Answer: (c)

36.    Tehri Hydro Development Corporation (THDC) India Ltd commissioned the first unit of its 400-Mw Koteshwar hydel project in March 2011.Koteshwar Hydro-Electric Project is located 22 Km downstream of Tehri Dam on which of the following rivers?
a.    Bhagirathi
b.    Alaknanda River
c.    Mandakini River
d.    Dhauliganga River
Answer: (a)

37.    According to the Engineering Export Promotion Council (EEPC) data by what percentage did engineering export grow in month of February 2011?
a.    72.3%
b.    75%
c.    75.7%
d.    77.4%
Answer: (b)

38.    Based on the recommendations of FIPB the Indian Government on 23 March 2011 approved 14 Foreign Direct Investment proposals and 27 proposals. What is the total worth of the FDI proposals accepted by the government?
a.    Rs 1289 crore
b.    Rs 1500 crore
c.    Rs 1342 crore
d.    Rs 1100 crore
Answer: (a)

39.    The finance ministry has opposed the Reserve Bank of India’s suggestion to restrict foreign direct investment in new banks. RBI had restricted foreign direct investment in new banks to what percentage?
a.    43%
b.    43.22%
c.    45.7%
d.    49%
Answer: (d)

40.    Consider the following statements-

1.    CAG in March 2011 sent a questionnaire consisting 40 questions to the telecom ministry on 2G spectrum allocation scam and other developments in the sector.
2.    The department of telecom (DoT) was asked as to why the spectrum (radio waves) allocation was not delinked from the licence as it was known that spectrum was scarce vis-a-vis the demand.
3.    The Parliamentary panel wanted to know whether DoT consulted telecom regulator TRAI regarding the “No Cap” recommendation on number of operators in a circle.
4.    The loss incurred by the exchequer as a result of the 2G scam is estimated to be Rs 185379 crore
Which of the above mentioned statements related to the 2G scam is true?
a.    All of the above statements
b.    1 & 4
c.    2 & 3
d.    2 & 4
Answer: (c)

41.    Which of the following bodies ordered restriction on transmitting unauthenticated news by brokers and wealth managers on blogs and mobile phones in an effort to prevent stock manipulation through rumours?
a.    RBI
b.    IRDA
c.    FIPB
d.    SEBI
Answer: (d)

42.    Which of the following bodies put off indefinitely the new rules governing unwanted telemarketing calls which were supposed to be implemented from 21 March 2011?
a.    Department of Telecommunication (DoT)
b.    Telecom Regulatory Authority of India (TRAI)
c.    Indian Ministry of communication & Technology
d.    HRD Ministry
Answer: (b)

43.    The Indian government in March 2011 conferred the Miniratna status on which of the following PSUs?
a.    National Small Industries Corporation (NSIC)
b.    Air India Charters Ltd
c.    Cement Corporation of India
d.    HMT Machine Tools Ltd
Answer: (a)

44.    Union Finance Minister Pranab Mukherjee on 11 March 2011 announced in Lok Sabha an increase in the allocation under the Members of Parliament Local Area Development Scheme (MPLAD) Scheme from Rs.2 crore to Rs.5 crore per Member. When was the MPLAD scheme introduced?
a.    1990
b.    1991
c.    1992
d.    1993
Answer: (d)

45.    Union Minister of Finance Pranab Mukherjee declared in the Lok Sabha on 11 March 2011 the extension of the existing Interest Subvention Scheme of providing short term loans to farmers at 7% interest with additional interest subvention for timely repayment to which of the following group?
a.    Small scale entrepreneurs
b.    Cottage industries & handicraft professionals
c.    Fishermen
d.    Horticulturists
Answer: (c)

46.    Indian agriculture needs Rs 108000 crore to fight climate change in the next five years to ensure food for all at a reasonable price by 2020. National Mission for Sustainable Agriculture is implemented under which of the following?
a.    NABARD
b.    Bharat Nirman
c.    Prime Minister's National Action Plan on Climate Change
d.    Accelerated Fodder Development Programme
Answer: (c)

47.    Read the following sentences with regard to NSIC.
1.    National Small Industries Corporation (NSIC) is a public sector undertaking under the ministry of micro, small and medium enterprises (MSME).
2.    NSIC was awarded the Mini Ratna status by the Indian Governmnet on 3 March 2011.
3.    NSIC has a marked presence both in the service as well as the manufacturing sectors.
4.    NSIC acts as a facilitator to promote small industries through various user-friendly and demand-driven schemes
Which of the following sentence/sentences is/are not true with regards to NSIC?
a.    Only 3
b.    1 & 3
c.    Only 2
d.    2 & 4
Answer: (a)

48.    In February 2011 imports rose to $31.70 billion, up 21.2 per cent over the same period in 2010 while exports rose 49.8 per cent to $23.60 billion. What was the trade deficit in February 2011?

a.     (-)8.1 billion
b.    (-)8 billion
c.    (-)8.3 billion
d.    (-)8.8 billion
Answer: (a)

496.    The Reserve of India on 17 March 2011 increased the interest rate at which it injects liquidity into the banking system by 25 basis points to 6.75 per cent (repo rate). It also revised its inflation projection for March-end 2011 from 7 % to _?

a.    7.3
b.    7.5
c.    7.5
d.    8
Answer: (d)

50. As per the official data released by TRAI on 9 March 2011, which state in MNP Zone-2 recorded the maximum number of request for mobile number portability by end of February 2011?
a.    Tamil Nadu
b.    Karnataka
c.    West Bengal
d.    Assam
Answer: (b) Karnataka

51. The Enforcement Directorate has booked Pune-based real estate agent and stud farm owner Hasan Ali Khan under section 3 of the Prevention of Money Laundering Act (PMLA), 2002. When did PMLA come into existence?
a.    2000
b.    2002
c.    2005
d.    1999
Answer: (c) 2005

52. The e-commerce portal of India Post, was launched by Union Minister Kapil Sibal on 9 March 2011 provide postal services online. Which among the following is the correct name of the portal?
a.    e-post office
b.    e-post service
c.    e-postal services
d.    India Post e-commerce
Answer: (a) e-post office

53. Steel tycoon Lakshmi Mittal overtook Mukesh Ambani to become the wealthiest Indian as per the annual Forbes list of World Billionaires for 2011. What is Lakshmi Mittal’s ranking in the list?
a.    Fourth
b.    Sixth
c.    Ninth
d.    Eleventh
Answer: (b) Sixth

54. According to the Economic Survey of India 2010-11, Forex Reserves estimated at __ US dollars.
a) 297.3 billion US dollars
b) 245.6 billion US dollars
c) 285.4 billion US dollars
d) 113.2 billion US dollars
Answer: (a) 297.3 billion US dollars

55. Consider the following statements regarding the Economic Survey of India 2010-11.
i) Agriculture likely to grow at 5.4% in 2010-11
ii) Trade Gap narrowed to 82.01 billion US dollars in April-December 2010
iii) 59% rise in Bank Credit
Please choose the right option:
a)    All the three statements  i, ii and iii are correct ; b) only I and ii are correct
b)    All three statements are wrong ; d) only i) is correct
Answer: (a) All three Statements i, ii and iii are correct

56.  India’s External debt stood at 295.8 billion US dollars at the end of September 2010, recording an increase of 33.5 billion US dollars ove the level of end-March 2010. The rise in debt was largely due to
i) Higher Commercial borrowings; ii) Short-term trade credits
iii) Multilateral government borrowings
Please choose the right option:
a)    Only i  b) only i and ii
b)    Only i and iii ; d) All I,ii and iii
Answer: (d) All i, ii and iii

57. Consider the following statements:
1. India’s exports crossed the USD 200 billion mark in the first eleven months of the 2010-11 fiscal.
2. The trade gap for the first 11 months of the financial year 2010-11 stood at USD 97.1 billion.
3. African countries and Japan were the traditional markets for Indian exporters during the 2010-11.
4. The current forecast for the 2011-12 is USD 230-235 billion.
Which of the statement given above is not correct?
a.1
b. 2
c. 3
d.4
Answer: (c) 3

58.    According to the Economic survey 2010-11 what percentage growth was recorded by the manufacturing sector in 2010?
a.    9.1%
b.    8.6%
c.    7.3&
d.    8%
Answer: (a)

59.    As per the economic Survey of India 2010-11, the production of food grains is estimated at over 232 million tonnes with record production of which food crop?
a.    Rice
b.    Wheat
c.    Bajra
d.    Maize
Answer: (b)

60.    What is the percentage growth of GDP predicted by The Economic Survey for the financial year 2010-11?
a.    8.6%
b.    8.3%
c.    8%
d.    9.1%
Answer: (a)

61.    The Economic Survey estimated the Forex reserves of India at over 297 billion US dollars. The surge in Forex is attributed to growth in which of the following sector?
a.    Export sector
b.    Foreign Direct Investments
c.    Agricultural output
d.    Industrial output
Answer: (a)

62.    Gross Fiscal Deficit stands at 4.8% of GDP. What was the percentage of Gross Fiscal Deficit in 2010?
a.    6.3%
b.    5.8%
c.    5%
d.    7.1%
Answer: (a)

63.    Agriculture is expected to grow by what percentage in 2010-11 as per the Economic Survey?
a.    5%
b.    5.1%
c.    5.4%
d.    5.5%
Answer: (c)

64.    What percentage of GDP growth at market prices was estimated by the Economic Survey 2010-11?
a.    9%
b.    9.5%
c.    9.7%
d.    10%
Answer: (c)

65.    Exports surged by what percentage in the period between April to December 2010?
a.    25%
b.    29.5%
c.    18.1%
d.    33%
Answer: (b)

66.    Finance Minister Pranab Mukherjee proposed to increase the Income Tax Exemption Limit for individual tax payers from 1 lakh 60 to _?
a.    1 lakh 80 thousand
b.    1 lakh 90 thousand
c.    2 lakh
d.    2 lakh 20 thousand
Answer:     (a)

67.    Which of the following was not proposed in the Union Budget 2011-12 presented by pranab Mukherjee?
1.    Special vehicles were proposed to be created in the form of Infrastructure Debt Funds to attract foreign funds.
2.    Rs. 300 crore expenditure was proposed to promote horticulture centres in rain fed areas for increasing crop productivity.
3.    For the manufacturing sector, the budget proposed reduction of basic customs duty on raw silk from 30 to 5 per cent.
4.    Concessional 10 per cent Excise Duty was also proposed for fuel cell or Hydrogen cell-technology-based vehicles.
a.    1 & 3
b.    Only 2
c.    Only 4
d.    3 & 4
Answer: (b)


68.    In the Budget it was proposed to provide sum of money for implementation of vegetable initiative to set in motion a virtuous cycle of higher production and incomes for the farmers. What was the proposed amount?
a.    Rs 500 crore
b.    Rs 300 crore
c.    Rs 10000 crore
d.    Rs 650 crore
Answer: (b)

69.    Rashtriya Swasthya Bima Yojana was proposed to be being extended to the beneficiaries of which of the following Union Government schemes?
a.    Mahatma Gandhi NREGA beneficiaries
b.    Beneficiaries of Swavlamban pension scheme
c.    Indira Gandhi National old Age Pension scheme beneficiaries
d.    Beneficiaries of Rajiv Gandhi Grameen Vidyutikaran Yojana
Answer: (a)

70.    Read the two statements mentioned with regard to the budgetary allocations for addressing environmental concerns.
1.    The budget proposed that the solar lantern used in far-flung villages will attract no duty from 10 per cent charged earlier.
2.    To provide green and clean transportation for the masses, National Mission for Hybrid and Electric Vehicles will be launched in the year 2011 in collaboration with all stakeholders.
Which of them is true?
a.    Only 1
b.    Only 2
c.    Both 1 & 2
d.    None of the above
Answer: (b)

71.    To enhance credit worthiness of economically weaker sections and LIG households, a Mortgage Risk Guarantee Fund was announced to be created under which of the following scheme or Yojana?
a.    Rajiv Awas Yojana
b.    Rajiv Gandhi Grameen Vidyutikaran Yojana
c.    Indira Awas Yojana
d.    Mahatama Gandhi NREGA
Answer: (a)

72.    Read the following statements with regard to the allocation in the Educational sector as proposed by Union Budget 2011-12.
1.    For Sarva Siksha Abhiyan the allocation was increased by 40 percent to 21000 crore rupees.
2.    All institutions of higher learning will be connected through optical fibers by March 2012.
3.    500 crore rupees was proposed to be provided for national skill development fund.
4.    For the needy scheduled castes ad scheduled tribe candidates studying in class-IX and Xth pre-matric scholarship scheme was proposed to be introduced.
Which of the above mentioned statements is not true?
a.    1
b.    2
c.    3 & 4
d.    4
Answer: (b)

73.    The Basic Customs Duty exemption was proposed to be extended to which of the following sectors?
a.    art and antiquities for exhibition or display in private art galleries
b.    Cinematographic film, factory-built ambulances
c.    syringes and needles
d.    agricultural machinery
Answer: (a)

74.    What amount of money was allocated for Bharat Nirman?
a.    1000 crore
b.    58000 crore
c.    55438 crore
d.    14362 crore
Answer: (b)

75.    The Union Budget for 2011-12 proposed lowering of qualifying age for tax relief for senior citizens from 65 years to _?
a.    55
b.    58
c.    60
d.    62
Answer: (c)

76.    Which of the following statement/statements is/are true with reference to the Railway Budget 2011?
1.    Railway Minister Mamata Banerjee proposed Annual Plan for the year 2011-12 at Rs 57630 cr which is the highest ever plan investment by the railways in a single year.
2.    Under the proposed Pradhan Mantri Rail Vikas Yojana the pending socially desirable lines would be completed and other similar new line projects would also be taken up.
3.    Mamata Banerjee declared 2010-11 as the Year of Green Energy.
4.    2 AC Double Decker Trains in the Jaipur-Delhi and Delhi-Ahmedabad routes were proposed.
Choose Answers:
a.    1,2 & 3
b.    1 & 4
c.    2 & 3
d.    1 & 2
Answer: (d)

77.    Acknowledging the need for development of J&K and North-east, Mamata Banerjee introduced in the Railway Budget 2011 a number measures for these two regions. In which of the following North-eastern cities did the budget propose to set up a diesel locomotive centre?
a.    Imphal
b.    Manipur
c.    Guwahati
d.    Tripura
Answer: (b)

78.    Read the two statements with regard to the passenger benefit plans proposed in the Railway Budget 2011.
1.    The budget proposed extension of Train Management System to New Delhi, Bangalore, Secunderabad, Ahmedabad and Lucknow stations to provide information on running of trains.
2.    Mamta Banerjee introduced a new concept of Smart Card - Go India for long distance travel by Indian railways.
Which of the two statements are true?
a.    Only 1
b.    Only 2
c.    Both 1 & 2
d.    None of the above
Answer: (c)

79.    According to Railway Budget 2011, railway card passes would be extended to the parents of the unmarried posthumous winners of which of the following Awards?
1.    Param Vir Chakra
2.    Bharat Ratna
3.    Ashok Chakra gallantry award
4.    Padma Shri
a.    1 & 2
b.    2 & 3
c.    1, 2 & 3
d.    1 & 3
Answer: (d)


80.    Which of the following schemes proposed in the Railway Budget 2011 is/are not meant for Railway Employees?
1.    Expansion of Liberalized Active Retirement Scheme for Guaranteed Employment
2.    Railway Vidyalaya Prabandhan Board
3.    Sukhi Griha Scheme
4.    Pradhan Mantri Rail Vikas Yojana
a.    1 & 3
b.    3 & 4
c.    2 & 4
d.    1 & 4
Answer: (b)

81.    The electrical energy requirement of railways is growing rapidly with the expansion of the rail infrastructure and traffic. Considering the rising demand the Ministry of Railways proposed to set up 700 MW power plant at Thakurli in Maharashtra. The power plant is proposed to be based on what form of energy?
a.    gas-based
b.    coal-based
c.    solar power
d.    nuclear energy based
Answer: (a)

82.    Ministry of Railways proposed to extend Anti Collision Device (ACD) to which of the following Railway zones in India
1.    Eastern zone
2.    East Central
3.    North Eastern
4.    South Central

a.    1 & 4
b.    2 & 4
c.    1 & 2
d.    1 & 3
Answer: (c)

83.    With regard to the infrastructure development of Railways as proposed in Railway Budget 2011 which statement/statements is/are false?
1.    The budget proposed to lay 40 new lines, covering 1075 km.
2.    The Ministry has allocated Rs 5406 crore for doubling of 867 km of lines
3.    A greater thrust was given to the expansion of the rail network with a larger allocation of Rs 9583 cr for new lines.
4.    for gauge conversion over 1017 km. Rs 13820 crore was proposed

a.    Only 1
b.    1 & 2
c.    Only 4
d.    2 & 4

Answer: (c)

84.    To raise money, the Railways was allowed for the first time, to issue tax-free bonds worth what sum of money?
a.    Rs 10000 crore
b.    Rs 5500 crore
c.    Rs 1000 crore
d.    Rs 5000 crore

Answer: (a)

85. For rural development allocation Union Budget 2011-12 is—
(A) Rs. 16,000 crore
(B) Rs. 46,000 crore
(C) Rs. 56,000 crore
(D) Rs. 87,800 crore

Answer: (d)

86. What is true for the service tax in Union Budget 2011-12 ?
(A) It is raised from 10 to 12%
(B) It is left unchanged at 11%
(C) It is left unchanged at 10%
(D) It is reduced from 14% to 12%

Answer: (c)

87. The target for exports in 2013-14 has been fixed at—
(A) $ 300 billion
(B) $ 275 billion
(C) $ 250 billion
(D) $ 450 billion

Answer: (d)

88. Global Hunger Index released by IFPRI in October 2010 places India at—
(A) 58th rank
(B) 64th rank
(C) 67th rank
(D) 74th rank

Answer: (c)

89. ‘Aam Admi Bima Yojana’ is an insurance scheme for rural landless households executed by the nodal agency—
(A) National Insurance Co.
(B) State Government
(C) LIC
(D) Central Government

Answer: (b)

90. Revenue Deficit as a per cent of GDP in Budget 2011-12 has been estimated at—
(A) 4.2%
(B) 6.8%
(C) 6.0%
(D) 4.6%

Answer: (d)

91. GST would be introduced from—
(A) January 1, 2012
(B) August 1, 2011
(C) April 1, 2012
(D) August 15, 2011

Answer: (c)

92. The rate of Minimum Alternate Tax (MAT) proposed in the budget 2011-12 is—
(A) 15%
(B) 18.5%
(C) 20%
(D) 22%
Answer: (b)

93. Which of the following is not a financial regulator ?
(A) IRDA
(B) AMFI
(C) PFRDA
(D) SEBI

Answer: (b)

94. Inflation in India is measured on which of the following indexes/indicators ?
(A) Cost of Living Index
(B) Consumer Price Index
(C) Wholesale Price Index
(D) Gross Domestic Product

Answer: (c)

95. As per 13th Finance Commission Recommendations during 2010-15, transfers to the states from the central tax pool are expected to be—
(A) Rs. 44000 crore
(B) Rs. 164832 crore
(C) Rs. 318581 crore
(D) Rs. 107552 crore

Answer: (c)

96. From which of the following taxes, the Central Government will get the maximum revenue in 2011-12 ?
(A) Custom Duties
(B) Income Tax
(C) Excise Duties
(D) Corporation Tax

Answer: (d)

97. As per quick estimates for the year 2010-11, Indian economy’s GDP at factor cost (at current prices) stood at—
(A) Rs. 3790063 crore
(B) Rs. 4713000 crore
(C) Rs. 4879232 crore
(D) Rs. 6426277 crore

Answer: (c)

98. What is the theme of World Development Report 2010 ?
(A) Poverty and Next Generation
(B) The Real Wealth of Nations : Path Ways to Human Development
(C) Incidence of Rural Poverty
(D) Development and the Next Generation

Answer: (b)

99. In New Direct Tax Code for senior citizens, income tax exemption slab has been raised to—
(A) Rs. 2.00 lakh
(B) Rs. 2.00 lakh
(C) Rs. 2.50 lakh
(D) Rs. 3.00 lakh

Answer: (c)

100. As per revised estimates for 2010-11 released by CSO, the growth rate for Indian economy has been estimated to be—
(A) 9.5%
(B) 8.6%
(C) 9.8%
(D) 6.7%

Answer: (b)

Tuesday, May 10, 2011

India Signs $69 Million Loan Agreement with ADB for Madhya Pradesh Power Sector Investment Programme

India and the Asian Development Bank (ADB) today signed a $69 million loan agreement which is the sixth and final tranche of the $620 million Madhya Pradesh Power Sector Investment Programme. This multi-tranche financing facility was approved by ADB on 29 March 2007. The signatories of the sixth tranche were Shri Venu Rajamony, Joint Secretary (Multilateral Institutions), Department of Economic Affairs, Ministry of Finance on behalf of the Government of India and Mr. Hun Kim, ADB’s Country Director for India.

The sixth tranche will assist the Madhya Kshetra Vidyut Vitaran Company reduce power transmission and distribution losses in 117 towns. It will also support the M. P. Poorv Kshetra Vidyut Vitaran Company, the M. P. Paschim Kshetra Vidyut Vitaran Company, and the M.P. Power Trading Company set up information technology (IT)-management systems linking all parts of the distribution and trading companies in the state. The IT management system will have a number of modules, including energy auditing, inventory tracking, and human resources and financial management, which will help the distribution companies increase efficiency. The sixth tranche will be implemented over a period of three and a half years.

The 6 tranches of the Madhya Pradesh Power Sector Investment Programme will increase transmission capacity in the state to about 10,000 megawatts (MW) from 5,563 MW and reduce transmission and distribution losses significantly. Enhanced power reliability, quicker repairs, along with better meter reading and billing, will also help improve the service to consumers. These measures will boost the financial performance of the state’s power sector and reduce its dependence on external support.

Speaking on the occasion, Shri Venu Rajamony said that long term impact of the programme will be inclusive economic growth of the people of the state, through enhanced power supply to households through reduced transmission and distribution losses, thereby increased transmission capacity. He said that the ADB program would improve operational efficiency in electricity distribution and financial sustainability of the distribution companies in the state.

Mr. Hn Kim, Country Director for India said that ADB has been a long-standing development partner for Madhya Pradesh. As of today, it has approved 14 loans amounting to $2.27 billion covering the power, urban, state road, and rural road sectors. ADB has also helped the state through a public resource management loan, Mr. Kim said.

Government Releases Funds for Power Development and Reforms Projects

The Ministry of Power has released Rs. 4029 Crores for taking up new projects under the Restructured Accelerated Power Development & Reforms Programme (R- APDRP). These projects will be taken up in two parts. While the Part-A will be for establishing IT enabled systems for energy accounting/ auditing and SCADA for big cities, Part-B will be for regular distribution upgradation and strengthening projects.

Out of the total released amount, Rs. 3903 Crores are in the form of loan for disbursement to state utilities while Rs. 126 Crores are as grant against enabling component for implementation of R-APDRP. Expressing satisfaction over the progress of the programme, the Union Power Minister, Shri Sushilkumar Shinde said here that all the States and Union Territories have invited Request for Proposals (RFPs) for appointing IT Implementing agencies for executing Part-A projects. So far 21 States and Union Territories have already appointed IT Implementing agencies.

The R-APDRP was launched in July, 2008 as a Central Sector Scheme with a total outlay of Rs. 51,577 Crores with an objective to reduce AT&C losses. The outlay for Part-A is Rs. 10,000 crore and for Part-B, it is Rs. 40,000 crore. The programme covers towns and cities with population more than 30,000 (10,000 for special category States).

While the Part-A projects worth Rs. 5177 Crores covering almost all the eligible 1401 towns have already been sanctioned, an additional 28 SCADA projects worth Rs. 669 Crores have also been sanctioned in six states including Gujarat, Andhra Pradesh, Kerala, Madhya Pradesh, Rajasthan and Tamil Nadu. Besides this, 823 projects worth Rs. 15,975 crores have been approved in 14 states including Andhra Pradesh, Gujarat, Himachal Pradesh, Karnataka, Kerala, Madhya Pradesh, Maharashtra, Punjab, Rajasthan, Sikkim, Tamil Nadu, Uttar Pradesh, West Bengal and Haryana under Part-B.

The use of Information Technology in distribution network through R-APDRP programme will help in establishment of complete energy auditing and accounting system, consumer indexing upto distribution transformer, facilitate meter reading, billing, collection, preparation of MIS, redressal of consumer grievances and establishment of IT enabled customer care centre.

Government's Plan, Programme & Policies

(1) Prerna:- The ‘janasankhya sthirata kosh (national population stabilization fund) has to promote & under take activities aimed at achieving population stabilization at a level consistent with the needs of sustainable economic growth. Social development and environment protection by 2070
Prerna is responsible for parenthood strategy. It is monetary incentive strategy aimed at pushing up the age of marriage of girls and delay the birth of the first child.
(2) National Rural Health Mission:-Launched in April 2005 the mission seeks to provide universal access to equitable, affordable and quality health care which is accountable and at the same time responsible to the needs of the people.
  • It also aims to achieve the goals set out under the national policy and the millennium development goals during the mission period.
(3) Rashtriya Swasthya Bima Yojna:-Launched by ministry of labour & employment, govt of India to provide health insurance coverage for BPL families.
  • Beneficiaries are entitled to hospitalization coverage up to Rs 30,000 for most of the disease that require hospitalization.
(4) National Food Security Mission:-Sponsored scheme launched in august 2007.
  • Objective is to increased production and productivity of wheat, rice and pulses.
(5) 15 Point Programme:-In Oct 2009 govt decided to include 3 more schemes in the Prime minister’s new 15 point programme for the welfare of minorities.
Those are: -
  • National rural drinking water programme.
  • Urban infrastructure developed scheme for small and medium town.
  • Urban infrastructure and governance scheme.
(6) Bharat Nirman Yojna:- It is a time bound business plan for action in rural infrastructure .Under Bharat nirman , action was proposed in the areas of:-
  • Irrigation.
  • Rural housing.
  • Rural water supply.
  • Rural electrification.
  • Rural telecommunication connectivity.
(7) National Mission On Education:-It is a mission in which education is provide through information and communication technology. “SAKSHAT” one stop education portal was launched on Oct 30, 2006 by the president of India.
  • Head of National knowledge commission:- Sam Pitroda.
(8) Right To Education Act 2009:- Article 21-(A), as inserted by the constitution (86th Amendment Act) 2002, provides for free and compulsory education of all children in the age group of 6 to 14 years as a fundamental rights. Consequently the parliament has enacted this in April 2009.
Salient features:-
(a) Free and compulsory education 6 to 14 age group.
(b) Will apply to all India except J&K.
(c) Provide for 25% reservation for economically disadvantaged communities in admission in private school.
(d) A child who completes elementary education (up to class 8) shall be awarded a certificate.
(9) Female Literacy:- It is a scheme to provide education & related facilities to ST Students launched by ministry of tribal affairs in December 11, 2009.
(10) Anil Kakodkar Committee on reforms in IITS:- it will suggest reforms to make these elite institutions a global brand.
(11) Yashpal Committee Report:- It was set up in 2008 for higher education and research.
  • It has suggested the scrapping of all higher education regulatory /monitoring bodies and creation of a super regulation.
  • It also recommended that the deemed university status be abandoned and that all deserving universities be either converted into full fledged universities or scrapped.
(12) National Rural Livelihood Mission:- Ministry of rural development and panchyati raj proposed to restructure the existing swarnjayanti gram swarojgar yojana into rural livelihood mission to have a focused approach to rural poverty eradication in a time bound manner.
  • Objective is to reduce poverty among rural BPL by promoting diversified and gainful self employment and wage employment opportunities which would lead to an appreciable increase in sustainable basis.
(13) NREGA Renamed After Mahatma Gandhi:- On Oct 2, 2009 Government has changed the name of National Rural Employment guarantee Act to Mahatma Gandhi National Rural Employment guarantee Act.
(14) Chandra Sheker Panel:- on the recomandation of a committee headed by cabinet secretary K.M Chander shaker. Centre had increased the pension for retired service man.
(15) Rajiv Awas Yojna:- Ministry of housing and urban poverty alleviation had launched the housing project called the Rajiv awas yojna for slum dwellers and the urban poor.
  • Aimed at making India slum free in the next five years.

Integrated Low Cost Sanitation (ILCS) Scheme

The “Integrated Low Cost Sanitation” Scheme basically aims at conversion of individual dry latrine into pour flush ones thereby liberating manual scavengers from the age old, obnoxious practice of manually carrying night soil.
The ILCS Scheme was initially started in 1980-81 through the Ministry of Home Affairs and later through Ministry of Social Justice and Empowerment. The scheme was transferred in 1989-90 to Ministry of Urban Development and Poverty Alleviation and from 2003-04 onwards to Ministry of UEPA/HUPA. The scheme has helped in constructing/converting over 28 lakh latrines to liberate over 60952 scavengers so far.
To make the Scheme more attractive and implementable the Guidelines have been revised with effect from 17th January 2008.
At the time of revision of guidelines of the scheme the State of Assam, Bihar, Jammu & Kashmir, Nagaland and Uttar Pradesh together showed six lakh dry latrines. Later the State of Assam, Nagaland and Jammu Kashmir have declared that they have no dry latrines in their States. These figures changed during the implementation, after the house to house survey of all municipal areas for identification. Presently, the State of Bihar, Uttar Pradesh and Uttarakhand have reported existence of dry latrine and funds have eventually been sought to eliminate these.
Presently, only states of Uttar Pradesh, Uttarakhand and Bihar have to declare themselves dry latrine free. The revised ILCS Scheme envisages conversion of all existing dry latrines within a period of three years (2007-2010). In the video conference held on September 28, 2010, with the representatives of States of Bihar, Uttarakhand and Uttar Pradesh, these states have assured that the conversion task will be completed by 31st December 2010 and they would be able to declare themselves dry latrine free.

Monday, May 9, 2011

Corporation Bank Probationary Officers Exam. 2011 Solved Paper

General Awareness on Socio-Economic Developments 

(Exam Held on 16-1-2011)

1. As per the decision taken by the Govt. of India now the National Rural Employment Guarantee Act is extended to all the districts of India. This means it will now be applicable to about—
(A) 200 districts
(B) 300 districts
(C) 400 districts
(D) 500 districts
(E) 626 districts
Ans : (E)

2. Which of the following is the amount of insurance cover provided to the workers of the unorganized sector under Rashtriya Swasthya Bima Yojana ?
(A) Rs. 10,000
(B) Rs. 20,000
(C) Rs. 30,000
(D) Rs. 40,000
(E) Rs. 50,000
Ans : (C)

3. Which of the following banks has taken over the Centurion Bank of Punjab ?
(A) ICICI Bank
(B) IDBI Bank
(C) HDFC Bank
(D) AXIS Bank
(E) None of these
Ans : (C)


4. Unique Identification Authority of India will set up data base for—
(A) Identity and biometrics details of Indian residents
(B) Identity and biometrics details of Indian citizens
(C) Identity and biometrics details of person’s residents in India or visiting India
(D) All the above
(E) None of these
Ans : (D)

5. The biggest Public Sector undertaking in the country is—
(A) Iron and steel plants
(B) Roadways
(C) Railways
(D) Airways
(E) None of these
Ans : (C)

6. Which of the following is Horticulture Crop ?
(A) Paddy
(B) Wheat
(C) Mango
(D) Bajara
(E) None of these
Ans : (C)

7. Which of the following nation is considered the originator of the concept of Micro Finance ?
(A) India
(B) Bangladesh
(C) South Africa
(D) USA
(E) None of these
Ans : (B)

8. Very often we read in news papers about 3G or 3rd Generation technology. This is a set of standards used for which of the following purposes ?
(A) To combat climatic changes
(B) Production of Nuclear Energy
(C) Film Production
(D) Mobile Telecommunications
(E) All of these
Ans : (D)

9. What is the full form of ‘ULIP’, the term which was in the news recently ?
(A) Universal Life and Investment Plan
(B) Unit Loan and Insurance Plan
(C) Universal Loan and Investment Plan
(D) Uniformly Loaded Investment Plan
(E) Unit Linked Insurance Plan
Ans : (E)

10. The main function of I. M. F. is to—
(A) Finance investment loans to developing countries
(B) Act as a private sector lending arm of the World Bank
(C) Help to solve balance of payment problems of member countries
(D) Arrange international deposits from banks
(E) None of these
Ans : (C)

11. The difference between visible exports and visible imports is defined as—
(A) Balance of trade
(B) Balance of payment
(C) Balanced terms of trade
(D) Gains from trade
(E) All the above
Ans : (A)

12. Invisible Export means export—
(A) Services
(B) Prohibited goods
(C) Unrecorded goods
(D) Goods through smuggling
(E) All the above
Ans : (A)

13. ‘Indira Gandhi Canal’ which is around 450 km long provide irrigation facility mainly to which of the following states ?
(A) Punjab
(B) Haryana
(C) Madhya Pradesh
(D) Gujarat
(E) Rajasthan
Ans : (E)

14. The rate of MAT has been increased from the present 15% to 18% with effect from April 2010. What is the full form of MAT ?
(A) Maximum Alternate Tax
(B) Minimum Alternate Tax
(C) Minimum Affordable Tax
(D) Maximum Affordable Tax
(E) None of these
Ans : (B)

15. RBI has asked banks to make a plan to provide banking services to all villages having a population upto 2000. This directive issued by the RBI will fall in which of the following categories ?
(A) Plan for Financial Inclusion
(B) Efforts to meet the targets of Priority Sector Lending
(C) Extension of Relief Packages to the Farmers
(D) Plan for opening more rural branches
(E) None of these
Ans : (D)

16. Which of the following is not a part of the scheduled banking structure in India ?
(A) Money Lenders
(B) Public Sector Banks
(C) Private Sector Banks
(D) Regional Rural Banks
(E) State Cooperative Banks
Ans : (A)

17. The rate of interest on Savings Bank Account is stipulated by—
(A) The concerned bank
(B) RBI
(C) Indian Banks Association
(D) Government of India
(E) Banking Codes and Standards Board of India
Ans : (B)

18. Many times we read a term CBS used in banking operation. What is the full form of the letter ‘C’ in the term ‘CBS’ ?
(A) Core
(B) Credit
(C) Continuous
(D) Complete
(E) None of these
Ans : (A)

19. In the year 2010, Commonwealth Games were organized in which of the following countries ?
(A) China
(B) South Africa
(C) Canada
(D) Brazil
(E) India
Ans : (E)

20. Which of the following metals is used for generation of nuclear Energy by most of the Nuclear Power Plants ?
(A) Zinc
(B) Platinum
(C) Uranium
(D) Nickel
(E) None of these
Ans : (C)

21. Many a time we read a term ‘PPP’ in financial dailies/magazines. What is the full form of the same as used in financial world ?
(A) Public Per Capita Power
(B) Per Capita Potential Purchases
(C) Purchasing Power Parity
(D) Present Purchasing Power
(E) None of these
Ans : (C)

22. The recent press report indicates that one of the Economies of the world took over another Economy in GDP terms to gain nubmer Two, ranking at world level. Which one of the following country is that ?
(A) Japan
(B) China
(C) Germany
(D) Australia
(E) India
Ans : (A)

23. Which of the following villagers received first lot of UID numbers ?
(A) Vaijapur in Maharashtra
(B) Bedkihal in Karnataka
(C) Tembhli in Maharashtra
(D) Navagam in Gujarat
(E) None of these
Ans : (C)

24. The highest weight in the revised Wholesale Price Index, implemented from September 2010 is given to which of the following item ?
(A) Fuel
(B) Food items
(C) Manufactured items
(D) Primary Articles
(E) All of these
Ans : (E)

25. Which one of the following is a driving force influencing the industrial growth of an economy ?
(A) Economic Factors only
(B) Investment only
(C) Innovation/Market Base only
(D) Only (A) and (B)
(E) All (A), (B) and (C)
Ans : (E)

26. What do you understand by Community Development programmes ?
(A) Villages participation in the programme
(B) A movement to promote better living for the whole community
(C) A movement to promote better living for the whole community with the assistance of local Government
(D) A movement designed to promote better living for the whole community with the active participation and on the initiative of the community
(E) None of these
Ans : (A)

27. In which one of the following activities percentage share of cooperative sector is the highest ?
(A) Agricultural Credit disbursement
(B) Sugar production
(C) Wheat procurement
(D) Fertilizer distribution
(E) Handlooms
Ans : (E)

28. What is a Panchayati Raj ?
(A) It is a community development programme
(B) It is a cooperative movement
(C) It is a scheme of self governance
(D) It is an exercise in decentralisation of administrative authority
(E) It is an out dated system, not followed now
Ans : (D)

29. Which one of the following is the updated base for Wholesale Price Index (WPI) ?
(A) 2002-2003
(B) 2003-2004
(C) 2004-2005
(D) 2005-2006
(E) 2006-2007
Ans : (C)

30. What is a Green Index ?
(A) It measures nation’s wealth according to GNP per capita
(B) It measures nation’s wealth according to GDP per captia
(C) It measures nation’s wealth according to provisions of control on Green house gases
(D) It measures nation’s wealth in terms of coverage of forest area
(E) None of these
Ans : (D)

31. In which one of the following city, 2012 Olympics are proposed to be held ?
(A) Frankfurt
(B) London
(C) Tokyo
(D) Washington
(E) Dubai
Ans : (B)

32. In which one of the following states, Rosa Power Plant is located ?
(A) U.P.
(B) M.P.
(C) Orissa
(D) Haryana
(E) Punjab
Ans : (A)

33. Which one of the following is present Saving Bank Interest Rate (in percentage terms) ?
(A) 3•0
(B) 3•5
(C) 4•0
(D) 4•5
(E) 5•0
Ans : (B)

34. For calculation of poverty line in rural area, which one of the following calories consumption per day per person has been stipulated by Planning Commission ?
(A) 2000
(B) 2100
(C) 2400
(D) 2500
(E) None of these
Ans : (C)

35. Who is Julian Assange ?
(A) Founder, Googles
(B) Founder, Rediffmail
(C) Nominee, Nobel Prize for Literature 2010
(D) Founder, WikiLeaks
(E) None of these
Ans : (D)

36. For which one of the following States, interloculators’ panel has been appointed by the Centre ?
(A) Manipur
(B) Arunachal Pradesh
(C) Assam
(D) Nagaland
(E) J&K
Ans : (E)

37. Expand the term BCBS.
(A) Bank’s Committee on Banking Supervision
(B) Basel Committee on Banking Supervision
(C) Bank’s Commission on Banking Supervision
(D) Basel Commission on Banking Supervision
(E) None of these
Ans : (B)

38. By which one of the following years, Government proposes to introduce IFRS norms ?
(A) April 2011
(B) October 2011
(C) April 2012
(D) October 2012
(E) None of these
Ans : (A)

39. As per World Development Report 2009, if the poverty is measured by the international poverty line standard of below a dollar a day, approximately what per cent of people will be living in India below poverty line ?
(A) 25.0
(B) 27.5
(C) 34.5
(D) 58.5
(E) 10.5
Ans : (B)

40. Who is Kaushik Basu ?
(A) Dy. Governor, RBI
(B) Member Planning Commission
(C) Chairman, Prime Ministers’ Economic Advisory Council
(D) Chief Economic Advisor
(E) Secretary-General FICCI
Ans : (D)

41. In a Poverty Pyramid index, which one of the following segments of population face highest poverty risk ?
(A) Formal wage employment
(B) Informal Self Employment : Employers
(C) Regular Informal Wage employment
(D) Informal Self Employment : own account
(E) Casual Informal Wage Employment and domestic work
Ans : (A)

42. Which one of the following is a Self Employment Programme for Educated Unemployed Youth ?
(A) Prime Minister’s Rozgar Yojana
(B) Swaran Jayanti Sahakari Rozgar Yojana
(C) National Social Assistance Programme
(D) Swaran Jayanti Gram Swarozgar Yojana
(E) None of these
Ans : (A)

43. In which one of the following States, Sariska Tiger Reserve is located ?
(A) M. P.
(B) Gujarat
(C) Rajasthan
(D) West Bengal
(E) Orissa
Ans : (C)

44. Who is Mohammed Yunus ?
(A) Finance Minister, Bangladesh
(B) Founder, Grameen Bank
(C) Nobel Laureate for peace
(D) Foreign Minister, Pakistan
(E) Founder Micro Finance Institutions in India
Ans : (B)

45. Which one of the following is the primary source of energy in India ?
(A) Hydel Power
(B) Natural Gas
(C) Coal
(D) Naptha
(E) Non-conventional energy
Ans : (C)

46. At which one of the following rate, the Central Bank lends to banks against government securities ?
(A) Repo Rate
(B) Reverse Repo Rate
(C) Bank Rate
(D) SLR
(E) CRR
Ans : (E)

47. Which one of the following is not a member of BASIC ?
(A) India
(B) China
(C) South Africa
(D) Brazil
(E) Indonesia
Ans : (E)

48. Which one of the following Corporate Groups has argued in Supreme Court that leaks of Radia Tapes has affected its reputation ?
(A) RIL
(B) Jindal Steels
(C) Bajaj Group
(D) Adani Group
(E) Tata Group
Ans : (E)

49. Yuan currency belongs to which one of the following countries ?
(A) Japan
(B) Thailand
(C) South Korea
(D) China
(E) Vietnam
Ans : (D)

50. Aung San Sau Kyi, Nobel Peace Prize Winner belongs to which one of the following countries ?
(A) China
(B) North Korea
(C) South Korea
(D) Japan
(E) Myanmar
Ans : (E)

Sunday, May 8, 2011

Census Of India 2011


The Indian Census is a credible source of statistical information on different characteristics of the citizens since 1872. This was conducted at different points of time in different parts of the country. In 1881 a Census was taken for the entire country simultaneously. Since then, Census has been conducted every ten years, without a break. The Census provides a snapshot of the country's population and housing at a given point of time. The Office of the Registrar General and Census Commissioner, India under the Union Ministry of Home Affairs is the nodal authority for conducting decennial Census in the country. Census 2011 is the 15th National Census of the country since 1872 and the 7th after Independence.


Importance of Census:
Census provides detailed and authentic information on demography, economic activity, literacy and education, housing & household amenities, urbanisation, fertility and mortality, scheduled castes and scheduled tribes, language, religion, migration, disability and many other socio-cultural and demographic data.
This information helps the Central and State Governments in planning and formulation of various policies. Besides, the delimitation or reservation of constituencies-Parliamentary/Assembly/Panchayats and other local bodies- is also based on demographic data.



Census of India 2011:
Census 2011 was conducted in two phases. The first phase, called the House Listing or Housing Census was conducted between April and September last year across the country, depending on the convenience of different States/UTs. The second phase, Population Enumeration, began simultaneously all over the country from February 9, 2011 and continued up to February 28, 2011.

 New Features of Census 2011:
Fresh Categories
The following fresh categories have been incorporated for comprehensive and better data:
  • Gender: New category "Other" introduced in addition to Male and Female.
  • Date of Birth question introduced along with Age.
  • Current Marital Status: Separate codes Assigned for Separated and Divorced.
  • New filter Question on SC/ST Introduced - "Is this person SC/ST?"
  • Disability: Household Schedule of Census 2011 attempts to collect information on eight types of disabilities as against five included in the Household Schedule of Census of India 2001. The information is being collected on disabilities namely, disability 'In Seeing', 'In Hearing', 'In Speech', 'In Movement', 'Mental retardation', 'Mental Illness', 'Any Other' and 'Multiple Disability'.
  • Literacy Status for "Other" sex added in addition to existing Male and Female.
  • New Codes under Status of Attendance in Educational Institutions introduced for Not Attending viz., (i) Attended before and (ii) Never attended.
  • Work: Marginal workers have been classified into two categories viz., (i) worked for 3 months or more but less than 6 months (ii) worked for less than 3 months. The definition of 'Main worker' remains the same.
  • A separate code-5 has been included under Non-economic activity for renters.
  • Migration - Provision to specify the present name of the Village/Town of the Birth Place as well as the Place of Last Residence introduced.
  • Name of the Institutional Household is also being recorded.
Census in Schools Census 2011 has taken initiatives to sensitise school students about census operations. The Census Organization is implementing "Census in School' programme across the country. This is specifically designed for the active participation of children in ensuring authenticity of census data of their families.
The programme covers about 60 to 80 schools in each of the 640 Districts in the country.

Mascot of an Enumerator
The mascot of an enumerator has been created for Census 2011 to make the process more people-friendly. The mascot helps people relate with the Census process and elucidates the key role of enumerators in the process. A toll free number 1800-345-0111 and services of call centre have been introduced for addressing public grievances.

Census Data
According to provisional results, India's population grew to 1.21 billion. The absolute number of children in the 0-6 age group recorded decline from 163 million in the 2001 census to 158 million in 2011. For detailed information click on the links given below:
Number of Administrative Units in Census 2011
  • State/Union Territories: 35
  • Districts: 640
  • Sub-districts: 5,924
  • Towns: 7,938
  • Villages: 6.41 Lakh
The cost of Census 2011 has been estimated at Rs 22,000 million, which works out to a per person cost of Rs.18.19. A total of 2.7 million functionaries worked in the conduct of the census. The census schedules were canvassed in 16 languages. A total of 340 million schedules were printed.

National Population Register (NPR) A milestone of Census 2011 is the creation of National Population Register (NPR). The National Population Register (NPR) will build up a comprehensive identity database of usual residents of the country. It would have the biometric data and UID Number of every person (15 years and above).
National Identity Cards will be given in a phased manner to all usual residents by the Office of the Registrar General and Census Commissioner, India.
The NPR is being introduced for the first time in the country. More on National Population Register
General Information about Census:
According to Article 246, population Census is a Union Subject in India. But, the State Governments provide administrative support in conducting the Census process.
The Office of the Registrar General and Census Commissioner, headed by the Registrar General and Census Commissioner, plans and implements Census. There are field offices, headed by Directors of Census Operations, in all the States and Union Territories (except Dadra and Nagar Haveli and Union Territory of Daman and Diu), which are attached to the office at Gujarat.
Directors of Census Operations are responsible for the conduct of Census in their respective jurisdiction.
The first step towards executing Census process involves preparation of a complete and unduplicated list of all geographical entities in the country, which include States, Districts, Tehsils/Taluks/ Community Development (CD) Blocks and Villages/Town at a particular point of time. More on Census Jurisdiction
The enumerators, supervisors and other field functionaries visit each and every household and ask questions to filling up Census forms. The information collected about individuals is kept absolutely confidential.
After the field work, the Census forms are transported to data processing centres located at 15 cities across the country. For faster data processing, the Intelligent Character Recognition Software (ICR) is used which saves a huge amount of manual labour and cost. The ICR technology with advanced features scans the Census Forms at high speed and extracts the data automatically. The Census is a statutory exercise conducted under the provisions of the Census Act, 1948 and Census Rules, 1990.
With a history of more than 130 years, this reliable, time tested exercise has been bringing out a veritable wealth of statistics. To scholars and researchers in demography, economics, anthropology, sociology, statistics and many other disciplines, the Indian Census has been a fascinating source of data. The rich diversity of the people of India is truly brought out by the decennial census which has become one of the tools to understand and study India.

 National Population Policy 2000:

The National Population Policy, 2000 (NPP 2000) affirms the commitment of the Government towards voluntary and informed choice and consent of citizens while availing of reproductive health care services, and continuation of the target free approach in administering family planning services. The NPP 2000 provides a policy framework for advancing goals and prioritizing strategies during the next decade, to meet the reproductive and child health needs of the people of India, and to achieve net replacement levels (TFR) by 2010. It is based upon the need to simultaneously address issues of child survival, maternal health, and contraception, while increasing outreach and coverage of a comprehensive package of reproductive and child health services by government, industry and the voluntary non-government sector, working in partnership. More on National Population Policy 2000

Important Points Of Food Security

For increasing the availability of food several steps have been taken such as the following:

  • Rashtriya Krishi Vikas Yojana with an outlay of Rs. 25000 crore.
  • Naitonal Food Security Mission with an outlay of about Rs. 6,000 crore.
  • National Horticulture Mission with an outlay of Rs 10,363.46 crore during the 11 the Five-Year Plan period.
  • There are many other schemes dealing with different areas of production, such as soil healthcare, crop protection, and irrigation. Inspite of all these schemes our agriculture is still very vulnerable to the behaviour of the monsoon.
  • Our country faces the challenge of producing food not only for 1.2 billion people, but also for about a billion farm animals.
  • Nearly seventy per cent of our population lives in villages and their main sources of livelihood are crop and animal husbandry, fisheries, agro-forestry, agro-processing and agri-business
  • The National Commission on Farmers (2004-06) has provided a detailed strategy for the agricultural progress of India.
  • Food is the first among the hierarchical needs of a human being. Therefore, food security should have the first charge on the available financial resources.
  • A National Food Security Act giving legal rights to food can be implemented only by attending to the safe storage of both grains and perishable commodities like fruits, vegetables and milk.

Indian Agri-Business:

Facts and Figure:-

  1. 2nd largest arable land (184 million hectares) in the world
  2. Largest irrigated land (55 million hectares) in the world
  3. Largest producer of wheat (72 million tones), accounting for nearly 15% of global wheat production
  4. Largest producer of pulses (15 million tones), accounting for nearly 21 % of global pulse production
  5. Largest producer of milk (90 million tones)
  6. Largest producer and exporter of spices
  7. Largest producer of tea, accounting for nearly 28% of the global tea production
  8. 2nd largest producer of rice (92 million tones), accounting for nearly 22% of global rice production.
  9. Largest producer of world's best basmati rice
  10. 2nd largest producer of fruits (50 million tones) and vegetables (100 million tones)
  11. 2nd largest producer of sugarcane (296 million tones), accounting for nearly 21 % of the global sugarcane production
  12. 3rd largest producer of coarse grains (31 million tones), including maize, accounting for nearly 4% of the global coarse grain production
  13. 3rd largest producer of edible oilseeds (25 million tones), accounting for nearly 7% of the global oilseed production
  14. Largest livestock population
  15. India produces 6.3 million tones offish (3rd largest in the world) Meat production is estimated at 2.3 million tones.

Progress under the national horticulture Mission:

  • Between 2005-06 to 2009-10, 2199 new nurseries were setup, additional area of about 16.56 lakh hectares was brought under new gardens of various horticultural crops and 2.78 lakh hectare of old and senile orchards was rejuvenated under the national Horticulture Mission. An area of 1.37 lakh hectare was covered under organic farming. Integrated Nutrient Management (INM) and Integrated Pest Management (IPM) were adopted in an area of 7.48 lakh hectares apart from setting up of 307 INM/IPM infrastructure which include 66 disease forecasting units, 78 bio control labs, 95 plant health clinics and 68 leaf/tissue analysis labs. Under Post Harvest Management Component, funds have been provided for setting up of 1328 pack houses, 343 cold storage units, 5 CA storage, 30 refrigerated vans, and 346 mobile/primary processing units. To ensure proper handling and marketing of horticulture produce, funds have been provided for setting up 32 whole sale markets and 298 rural markets. 7.74lakh farmers have been trained under various horticultural activities.
  • A web enabled progress monitoring system has been put in place to monitor progress under NHM on monthly basis. The system enables online uploading of physical and financial progress for each of the activities approved for implementation as per the Annual Action Plan at the state level as well as at the district level on the NHM web site. The System also facilitates the generation of different types of reports at national, state and district levels for the purpose of analysis and review.
  • AGRICULTURE in India still accounts for 52% of employment .12 % of national export and 17.8% of GDP.

Cabinet Clears Caste Census In 2011

After several deliberations within the government on the politically-sensitive issue, the Union cabinet decided to carry out a separate house-to house caste enumeration from June to September next year.
Giving in to demands from almost all political parties, government on Sep 9 2010. decided to hold caste census next year under an exercise to be carried out separately from the enumeration of population.
After several deliberations within the government on the politically-sensitive issue, the Union cabinet decided to carry out a separate house-to-house caste enumeration from June to September next year.
According to home minister P. Chidambaram, after considering various options, the option that we have approved is, based on the responses of various political parties, that caste must be canvassed and the integrity of the headcount must not be affected. The caste enumeration will be conducted in a phased manner after the population enumeration, which will include biometric capture and headcount, is completed by March next year, he said. He evaded a reply when asked whether caste enumeration would be merged with the headcount.
The decision has been taken after considering options suggested by the Group of Ministers (GoM) set up on the issue and the consultations that finance minister Pranab Mukherjee held with leaders of various parties.
Parties like Rashtriya Janata Dal, Samajwadi Party and Janata Dal (United) had disrupted proceedings in Parliament for several days during the Budget and Monsoon sessions while pressing for caste census.
The Bharatiya Janata Party, which was divided initially, later decided to support it. The issue even divided the Congress as well as the Union cabinet with some favouring caste census and others opposing it, prompting the government to set up a GoM to consider all aspects related to such a move.
Chidambaram said a suitable legal regime for collection of data on castes would be formulated in consultation with the ministry of law and justice.
There will be an additional cost for the exercise which will be assessed at a separate meeting. The office of the Registrar General and Census Commissioner would conduct the field operations of the caste enumeration.
The Central government will constitute an expert group to classify the caste/tribe returns after the enumeration is completed. The office of the Registrar General and Census Commissioner would hand over the details of the castes/tribes returned in the enumeration to the proposed expert group.
The last caste-wise census was held in 1931 and such a practice had been given up as a matter of policy after Independence.
In response to the demands for enumerating castes other than Scheduled Castes and Scheduled Tribes in the Census 2011 raised inside Parliament as well as by various groups outside, the ministry of home affairs submi 7ed a note to the Union cabinet in May 2010 pertaining to this issue.

India launches Census 2011, the biggest-ever in history

India on Apr 01, 2010 launched “Census 2011,ftft the biggest-ever census a7empted in the history of mankind enumerating the country's 1.2 billion population and classifying usual residents according to their gender, religion, occupation and education.
The massive exercise, to be spread over next 11 months, will mark a milestone as the first-ever National Population Register (NPR) will also be prepared in which all persons aged over 15 years will be photographed and fingerprinted to create a biometric national database. With this India will probably become the first democratic nation in the world which would have got its population fingerprinted in a year from now.
As the first citizen of the country, President Pratibha Devisingh Patil was the first person to be listed in the decennial exercise. She appealed to her compatriots to follow her example “for the good of the nation.
The 15th National Census exercise, since 1872, will see over 25 lakh officials capturing the socio economic cultural profile of its citizens. It will cost around Rs. 2,209 crore while the expenditure on NPR will be Rs. 3,539.24 crore. The exercise will also consume more than 11 million tonnes of paper.
During the massive exercise, the enumerators for the first time will collect information like ownership of mobile phones, computers, internet, having treated or untreated drinking water facility and usage of banking services. They will also seek additional information for the creation of NPR. The government has already said that no information will be collected on castes as no caste-based census has ever been conducted in independent India.
The second phase, called the Population Enumeration phase, will be conducted simultaneously all over the country from February 9 to 28, 2011, and the entire exercise would be completed by March 5, 2011.
All 640 districts, 5,767 tehsils, 7,742 towns and more than six lakh villages will be covered.

What is Census

The Indian Census is the most credible source of information on Demography (Population characteristics), Economic Activity, Literacy & Education, Housing & Household Amenities, Urbanization, Fertility and Mortality, Scheduled Castes and Scheduled Tribes, Language, Religion, Migration, Disability and many other socio-cultural and demographic data since 1872. Census 2011 will be the 15th National Census of the country. This is the only source of primary data at village, town and ward level. It provides valuable information for planning and formulation of polices for Central & State Governments and is widely used by National & International agencies, scholars, business people, industrialists, and many more. The delimitation/reservation of Constituencies.
Parliamentary/Assembly/Panchayats and other Local Bodies is also done on the basis of the demographic data thrown up by the Census. Census is the basis for reviewing the country's progress in the past decade, monitoring the on-going schemes of the Government and most importantly, plan for the future. That is why the slogan of Census 2011 is "Our Census, Our Future".

National Population Register

The NPR would be a Register of usual residents of the country. The NPR will be a comprehensive identity database that would help in be7er targeting of the benefits and services under the Government schemes/programmes, improve planning and help strengthen security of the country. This is being done for the first time in the country.
How will both these exercises be conducted?
The Census is a statutory exercise conducted under the provisions of the Census Act 1948 and Rules made there under. The NPR is being created under the provisions of the Citizenship Act and Rules.

Census Process

The Census process involves visiting each and every household and gathering particulars by asking questions and filling up Census Forms. The information collected about individuals is kept absolutely confidential. In fact this information is not accessible even to Courts of law. After the field work is over the forms are transported to data processing centres located at 15 cities across the country. The data processing will be done using sophisticated software called Intelligent Character Recognition Software (ICR). This technology was pioneered by India in Census 2001 has become the benchmark for Censuses all around the globe. This involves the scanning of the Census Forms at high speed and extracting the data automatically using computer software. This revolutionary technology has enabled the processing of the voluminous data in a very short time and saving a huge amount of manual labour and cost.

NPR Process

Details such as Name, Date of Birth, Sex, Present Address, Permanent Address, Names of Father, Mother and Spouse etc will be gathered by visiting each and every household. All usual residents will be eligible to be included irrespective of their Nationality. Each and every household will be given an Acknowledgement Slip at the time of enumeration. The data will then be entered into computers in the local language of the State as well as in English. Once this database has been created, biometrics such as photograph, 10 fingerprints and probably Iris information will be added for all persons aged 15 years and above. This will be done by arranging  camps at every village and at the ward level in every town. Each household will be required to bring the Acknowledgement Slip to such camps. Those who miss these camps will be given the opportunity to present themselves at permanent NPR Centres to be set up at the Tehsil/Town level. In the next step, data will be printed out and displayed at prominent places within the village and ward for the public to see. Objections will be sought and registered at this stage. Each of these objections will then be enquired into by the local Revenue Department Officer and a proper disposal given in writing.
Persons aggrieved by such order have a right of appeal to the Tehsildar and then to the District Collector. Once this process is over, the lists will be placed in the Gram Sabha in villages and the Ward Commi7ee in towns. Claims and Objections will be received at this stage also and dealt with in the same manner described above. The Gram Sabha/Ward Commi7ee has to give its clearance or objection within a fixed period of time after  which it will be deemed that the lists have been cleared. The lists thus authenticated will then be sent to the Unique Identity Authority of India (UIDAI) for de-duplication and issue of UID Numbers. All duplicates will be eliminated at this stage based on comparison of biometrics. Unique ID numbers will also be generated for every person. The cleaned database along with the UID Number will then be sent back to the Office of the Registrar General and Census Commissioner, India (ORG&CCI) and would form the National Population Register. As the UID system works on the basis of biometric de-duplication, in the case of persons of age 15 years and above (for whom biometrics is available), the UID Number will be available for each individual. For those below the age of 15 years (for whom biometrics is not available), the UID Number will be linked to the parent or guardian.

Will Caste Census in India Change Anything?

The recent approval by the Government of India for caste based census has aroused concerns among political parties as well as social scientists and ethnographers. For the self-proclaimed ‘modernists’ such regressive measures are likely to heighten caste consciousness among Indians. While the pragmatists argue that data on caste can help the government to be7er target affirmative action policies and thereby address caste differences rationally. The fact however is the that caste as a social reality in India draws succour from multiple sources which will remain untouched by either the proposed census or the resultant official policies. Caste issues influence everyday life of Indians in countless ways.
For those who think that caste in India is merely a tool of political opportunism or a remnant of ancient Hindu culture, visit to the rural regions of North Kerala during the winter months can be surprising and unnerving. Every year as the winter sets in, rural areas of North Kerala prepare for a unique transformation of social relations. Theyyam is an art form where performers, belonging to lower castes, are believed to be incarnated by local deities and members of higher castes flock to seek their blessing. For three months of the year individuals from the lower castes are elevated to the status of God but have to live as untouchables once the Theyy am season culminates.
Numerous rituals and customs like Theyyam continue to nourish the practice of caste differences in India. One only needs to skim through the matrimonial columns in leading English dailies to get a sense that caste is an important consideration even for the ‘modern’ elite segment of the urban Indian population.
Stephen P. Rosen in his book Societies and Military Power: India and its Army discuss the continuing salience of caste in India’s army. Though many would consider Rosen’s conclusions somewhat extreme but he does raise several valid concerns. The ba7alions in the Indian Army continue to be organised as the Jat, Sikh, Dogra or Rajput regiments.
The Poona Pact between Mahatma Gandhi and B.R. Ambedkar in 1932 granting reservation for ‘depressed’ classes in the provincial legislatures was a precursor for the phenomenal political salience of caste in Indian politics. From the Backward Classes Commission’s endeavour to create a master list of other  backwards classes in 1955 to KHAM (Kshatriyas, Harijans, Adivasis and Muslims) politics of the Congress (I) in the 1980s; from the Mandal Commission uproar in the 1990s to growing mass appeal of Dalit based political par- ties like the Bahujan Samaj Party, India’s political arena has witnessed countless caste inspired stirrings. No observer of Indian politics can ignore the role of caste in determining electoral outcomes.
Moreover, even before the on-going controversy over the 2010 caste census erupted, the Government of India had been involved in many projects categorizing its population along caste lines. Despite criticisms from historians, political scientists and anthropologists the Government of India did not abandon the colonial state’s practice of differentiating people along lines of caste and religion in the People of India projects. According to Susan Bayly, in the late 1990s the Anthropological Survey of India was undertaking “massive exercise in caste-based data collection- the People of India Project- with funding from Planning Commission. The project has used DNA sampling to identify difference between individual castes and tribes. Readers are told that as composite ‘type’, the Scheduled Castes have ‘relatively broad noses’; chamars says another entry are characterized by a ‘long, narrow head shape and a long moderately broad nasal shape.” Such, outrageous distinctions based on caste in a report sponsored by a State agency had gone completely unnoticed by the ‘modernists’ and pragmatics involved in recent debate. Though previous censuses have not included questions of caste directly, the ethnographic notes in the census reports have contributed much to literature on caste in India.
According to the pragmatists India cannot simply assume ‘modernity’ by ignoring caste. They see caste as a form of social stratification much like class in Britain or race in the U.S. The U.S. census and job applications have a voluntary disclosure segment requiring information about the individual’s race. Comparing caste with class and race appears incorrect if one realises that unlike other forms of social stratifications, caste in India is allegedly sanctioned by the sacred scriptures. Even though the scriptural sanctity of caste is open to debate many Indians cite the Manusmriti and Bhagavad Gita to support arguments in favour of caste. It is much simpler to address differentiations based on skin colour or nationality than stratifications perceived to be ordained by religious texts.
Caste based social practices in India go beyond the more visible political and economic dimensions and these subtle aspects are unlikely to be impacted by the enumeration of caste through the census.

Brief History of Census

The earliest literature 'Rig-Veda' reveals that some kind of population count was maintained in during 800-600 BC in India. The celebrated 'Arthashastr' by 'Kautilya' wri7en in the 3rd Century BC prescribed the collection of population statistics as a measure of state policy for taxation. It contained a detailed description of methods of conducting population, economic and agricultural censuses. During the regime of the Mughal king Akbar, the administrative report 'Ain-e-Akbar' included comprehensive data pertaining to population, industry, wealth and many other characteris tics.
A systematic and modern population census, in its present form was conducted non synchronously between 18ti5 and 1872 in different parts of the country. This effort culminating in 1872 has been popularly labeled as the first population census of India However, the first synchronous census in India was held in 1881. Since then, censuses have been undertaken uninterruptedly once every ten year.
The Census of India 2001 was the fourteenth census in the continuous series as reckoned from1872 and the sixth since independence. The gigantic task of census taking was completed in two phases. In the first phase, known as House -listing Operations, all building and structures, residential, partly residential or non- residential were identified and listed and the uses to which they were put recorded. Information on houses, household amenities and assets were also collected. In the second phase, known as Population Enumeration, more detailed information on each individual residing in the country, Indian national or otherwise, during the enumeration period was collected.
At the Census 2001, more than 2 million (or 20 lakh) enumerators were deployed to collect the information by visiting every household. The Indian Census is one of the largest administrative exercises undertaken in the world.

Important Points Of HRD

  • The higher education System in India at present comprises 504 university and university-level institutions which include 243 state universities, 53 state private universities, 40 central universities.
  • At the commencement of the 11 the Five Year Plan, there were 19 Central Universities, 7 Indian Institutes of Technology (UTs), 20 National Institutes of Technology (NITs), 4 Indian Institutes of Information Technologies (IIITs), 2 Indian Institutes of Science Education and Research (USERs), 6 Indian Institutes of Management (UMs) and 1 School of Planning & Architecture (SPA), apart from few other centrally funded educational institutions.
  • In 2007-08 the share of university & higher education and technical education was 11.83% and 5.33 respectively of the total expenditure on education.
  • The share of technical education has increased from 3.8% in 2006-07 to 5.33% in 2007-08.
  • At the beginning of the academic year 2009-2010, the total number of students enrolled, in the formal system, in the Universities and Colleges has been reported at 136.42 lakhs. This enrolment translates into a Gross Enrolment ratio of about 12.9%. While the world average of the Gross Enrolment Ratio in higher education is 26.7%, the average of the developed countries is 57.7% and that of the developing countries is 13%. Thus, many more universities and colleges will need to be opened to improve the Gross Enrolment Ratio to a respectable level.
  • In case the GER in higher education is to increase to 30% by 2020, either the capacity of existing institutions has to be increased by approximately 200% or the number of institutions has to be increased additionally by double the number of existing institutions
  • The Yashpal Committee and also the National Knowledge Commission (NKC) have dealt with various issues affecting the higher education system in the country and both have suggested definite framework for improvement by way of institutional as well as policy reforms. One of the main suggestions is establishment of an overarching regulatory body, namely National Commission on Higher Education and Research, which would subsume the functions of existing regulatory institutions like University Grants Commission (UGC), All India Council for Technical Education (AICTE) and National Council for Teacher's Training.
  • Based on the recommendations and suggestions of the YCR and NKC, the central government has initiated the process for (i) establishment of educational tribunals; (ii) to provide for prohibition of certain unfair practices in technical and medical educational institutions and universities, (iii) to provide for mandatory accreditation of higher educational institutions and to create a regulatory authority for the purpose and (iv) to provide for regulation of entry and operation of foreign educational institutions. In addition, a Task Force constituted by the Ministry of Human Resource Development has developed a framework for the establishment of National Commission for Higher Education and Research
  • Indian economy in terms of purchasing power parity with an equivalent GDP of US$ 3.666 trillion is the fourth largest economy in the world after USA, China and Japan. In US Dollar Terms, it is the twelfth largest in the world, with a GDP of US $719.8 billion.
  • The Indian economy is expected to expected to grow with more than or about five percent growth till 2050 as projected by BRIC Report (2003).
  • According to the Eleventh Five Year Plan Document, only 2% of existing workforce in India has skill training, while the corresponding figures are 96%, 80% and 75% respectively for Korea, Japan and Germany.
  • As per the 2001 census India has a BPL population of roughly 260 million and 300 million school dropouts in the age bracket of 6 to 16.
  • In this country with the youngest population, nearly 13 million people enter the market every year.

Initiatives taken by the government of India:-

Coordinated Action on Skill Development:-
  • Coordinated Action on Skill Development is the major initiative for achieving 11 the Plan objective of inclusive growth through coordination and harmonization of skill development initiatives of different players. The action aims at creation of a pool of skilled manpower with adequate skills that meet the employment requirement across various sectors of the l1ational economy.
The main functions of the Council are:-
  • To lay down overall policy objectives, financing and governance models and strategies relating to skill development.
  • To review the progress of schemes, and guide on midcourse corrections, additions and closure of parts or whole of any particular programmed / scheme.
  • Coordinate Public Sector private Sector Initiates in a framework of a collaborative action.
  • The Council has set a target of creating 500 million skilled persons by 2022 with emphasis on inclusion so as to deal with divides of gender, rural/urban, organized/unorganized, employment and traditional contemporary work place.
National Skill Development Corporation:-
  • The National Skill Development Corporation (NSDC); a non profit company under Section 25 of the companies Act, has been set up under the Ministry of finance it has an equity base of Rs 10 crore which 49% is contributed by the Govt. and 51 % by the private sector.

Community information initiatives in India

Name of the Project Coverage Institutions Involved
Akshaya Karla State Information Technology Mission
Bhoomi Karnataka Revenue Dept., National Informatics Centre
Community Information Centres  North Eastern States and Jammu & Kashmir Ministry of Development of North Eastern Region, National Informatics Centre
Digital Gangetic Plane Uttar Pradesh Media Lab Asia, IIT- Kanpur
Drishtee Haryana, Punjab, Madhya Pradesh, Rajasthan, Bihar Drishtee Ltd.
E-choupal Madhya Pradesh, Andhra Pradesh, Karnataka, Maharashtra, Rajasthan International Business Division of ITC Ltd.
Gyandoot Madhya Pradesh Gyandoot Samiti, National Informatics Centre
Rural e-seva Andhra Pradesh West Godavari District Administration
TARAhaat Punjab, Haryana, Madhya Pradesh, Uttar Pradesh Developmental Authorities
Village Knowledge Centres 600,000 Villages in India in 2007 Government Agencies and NGOs

Global Competitiveness Report

India has slipped by two places to 51st in the World Economic Forum's global competitiveness rankings, while rival China has managed to improve its standing to 29th. As per the WEF's Global Competitiveness Report 2010-11, released on Sept 9, 2010 switzerland is No. 1 in the world in terms of its ability to provide the most competitive environment on several fronts.
Sweden, another technology powerhouse in Europe, ranks second, followed by Singapore and the United States, which both fell by two positions from their ranking last year. The African nation of Chad figures at the bottom of the list of 139 countries.
The global competitiveness rankings are viewed as a barometer of the business climate in 139 countries and mirrors the assessments of leading businessmen on a range of political, social, and economic parameters.
Though Switzerland has "[state-supported] monopolies in key sectors, it maintains overall economic stability and largely open trade and investment policies," said Margareta Dryeniek Hanouz, senior economist and director of the WEF, who is also the co-author of the report.
India has been pushed down to 51st position from 49th due to its poor performance in a range of social sector areas such as education, health and infrastructure.
Though India has performed well in complex financial sector areas, attaining the 17th rank globally in terms of its financial markets, 44th in business sophistication and 39th in innovation, it has failed to improve the basic drivers of competitiveness, the report said.
Life expectancy is 10 years shorter in India as compared to China and Brazil. Despite high economic growth, India continues to be plagued by budget deficits, high public debt and high inflation. In contrast, China has over USD 2 trillion in forex reserves and a sound macro-economic environment.
TheWEF, which is a non-governmental organisation, is largely known for its annual Davos show of captains of industry and business and political leaders. In the face of a growing economic crisis in the western world, the WEF has increasingly promoted "compassionate capitalism" as an economic model, analysts said.

Global Competitiveness Report

The Global Competitiveness Report is a yearly report published by the World Economic Forum. The first report was released in 1979. The 2009-2010 report covers 133 major and emerging economies, down from 134 considered in the 2008-2009 report as Moldova was excluded due to lack of survey data.
Switzerland leads the ranking as the most competitive economy in the world, as the United  States, which ranked first for several years, fell to fourth place due to the consequences of the financial crisis of 2007–2010 and its macroeconomic stability. China continue its relative rise in the rankings reaching 27th.
The report "assesses the ability of countries to provide high levels of prosperity to their citizens. This in turn depends on how productively a country uses available resources. Therefore, the Global Competitiveness Index measures the set of institutions, policies, and factors that set the sustainable current and medium term levels of economic prosperity."

Description

Somewhat similar annual reports are the Ease of Doing Business Index and the Indices of Economic Freedom. They also look at factors that affect economic growth, but not as many as the Global Competitiveness Report.
One part of the report is the Executive Opinion Survey which is a survey of a representative sample of business leaders in their respective countries. Respondent numbers have increased every year and is currently just over 11,000 in 125 countries.
The report ranks the world's nations according to the Global Competitiveness Index. The report states that it is based on the latest theoretical and empirical research. It is made up of over 90 variables, of which two thirds come from the Executive Opinion Survey, and one third comes from publicly available sources such as the United Nations. The variables are organized into nine pillars, with each pillar representing an area considered as an important determinant of competitiveness.
The report notes that as a nation develops, wages tend to increase, and that in order to sustain this higher income, labor productivity must improve in order for the nation to be competitive. In addition, what creates productivity in Sweden is necessarily different from what drives it in Ghana. Thus, the GCI separates countries into three specific stagesti factor-driven, efficiency driven, and innovation-driven, each implying a growing degree of complexity in the operation of the economy.
In the factor-driven stage countries compete based on their factor endowments, primarily unskilled labor and natural resources. Companies compete on the basis of prices and sell basic products or commodities, with their low productivity reflected in low wages.
To maintain competitiveness at this stage of development, competitiveness hinges mainly on well-functioning public and private institutions (pillar 1), appropriate infrastructure (pillar 2), a stable macroeconomic framework (pillar 3), and good health and primary education (pillar 4). As wages rise with advancing development, countries move into the efficiency-driven stage of development, when they must begin to develop more efficient production processes and increase product quality. At this point, competitiveness becomes increasingly driven by higher education and training (pillar 5), efficient markets (pillar 6), and the ability to harness the benefits of existing technologies (pillar 7).
Finally, as countries move into the innovation-driven stage, they are only able to sustain higher wages and the associated standard of living if their businesses are able to compete with new and unique products. At this stage, companies must compete by producing new and different goods using the most sophisticated production processes (pillar 8) and through innovation (pillar 9). Thus, the impact of each pillar on competitiveness varies across countries, in function of their stages of economic development. Therefore, in the calculation of the GCI, pillars are given different weights depending on the per capita income of the nation. The weights used are the values that best explain growth in recent years For example, the sophistication and innovation factors contribute 10% to the final score in factor and efficiency-driven economies, but 30% in innovation-driven economies. Intermediate values are used for economies in transition between stages.

2010-2011 rankings

The following are the top 30 countries in the 2010-2011 Report.
1. Switzerland 5.63
2. Sweden 5.56
3. Singapore 5.58
4. United States 5.43
5. Germany 5.39
6. Japan 5.37
7. Finland 5.37
8. Netherlands 5.33
9. Denmark 5.32
10. Canada 5.30
11. Hong Kong SAR 5.27
12. United Kingdom 5.25
13. Taiwan 5.21
14. Norway 5.14
15. France 5.13
16. Australia 5.11
17. Qatar 5.10
18. Austria 5.09
19. Belgium 5.07
20. Luxembourg 5.05
21. Saudi Arabia 4.95
22. South Korea 4.93
23. New Zealand 4.92
24. Israel 4.91
25. United Arab Emirates 4.89
26. Malaysia 4.88
27. China 4.84
28. Brunei 4.75
29. Ireland 4.74
30. Chile 4.69

Important Points Of Inflation

The causative factors at the initial stages were the sudden boom in global commodity--12rices which transmitted to the Indian economy.
  • The gradual accumulation of steam in inflation was now again led by commodity prices, but this time the push came from domestic causes. The government to its credit over the last few years has pumped in additional purchasing power in rural India through the NREGA and other better directed schemes. So the impoverished ends of villages are now demanding a better life chance, meaning more food. This is one of the reasons why food inflation refuses to go away.

Inflation Basics:

Inflation may be caused due to several economic factors:

  • When the government of a country prints money in excess, prices increase as there is too much money in circulation chasing too few goods.
  • Increase in production and labor costs have a direct impact on the price of the final product, resulting in inflation.
  • When countries borrow money they have to cope with the interest burden. This interest burden may result in inflation.
  • High taxes on consumer products can also lead to inflation.
  • Demand pull inflation is when the economy demands more goods and services than what is produced.
  • Cost push inflation or supply shock inflation is when non availability of a commodity would lead to increase in prices.

The problems due to inflation would be:

  • When the balance between supply and demand goes out of control, consumers could change their buying habits forcing manufacturers to cut down production.
  • Inflation can create major problems in the economy. Price increase can worsen poverty, affecting low income household,
  • Inflation creates economic uncertainty and is a dampener to the investment climate, slowing growth and finally reducing savings and thereby cutting consumption.
  • The producers would not be able to control the cost of raw material and labor and hence the price of the final product. This could result in less profit or in some extreme case no profit, forcing them out of business.
  • Manufacturers would not have an incentive to invest in new equipment and new technology.
  • Uncertainty would force people to withdraw money from the bank and convert it into product with long lasting value like gold, artifacts.

Migration Food Inflation

  • Food commodity driven inflation has become a persistent phenomenon and the corrective measures involve concerted efforts over in extended period of time with public investments already hiked substantially in the recent years, there is now a need to pay attention to improve efficiency of such investment As area available agriculture and food production is going to shrink.
  • Focus should be given enhancing the productivity of crops to keep pace with growing demand for food.
  1. Focus should be given on enhancing the productivity of crops to keep pace with growing demand for food.
  2. Overhaulting the PDS is another corrective measure which can be undertaken in the medium term.
  3. Piling up of food grains in the granary beyond the stipulated levels is an avoidable proposition. Resorting to open market sales at specific intervals would help in both relieving the granary of excess stocks and checking the build up of rices in the domestic market simmultaneously.
  4. It is also logical to widen the scope of PDS by including more essential commodities like pulses, edible oils and sugar, to provide some protection to the poor against food inflation.
  5. However, it is not clear wheather expansion in PDS coverage to inflation or it is adding to food inflation because of rising in subsidies and leakages in our delivery system.
  6. Resorting to food imports can help in checking domestic prices to make up the supply shortfalls, provided imports are plannes on time. Our past experiences with wheat imports turning costlier with international prices moving up with India's decision to go for import. We need to develop a system for getting advance information on demand and supply imbalances and tune our trade policy accordingly.
  7. As an immediate remedy, all steps to prevent hoarding and speculation in food commodities have to be expedited.
  8. The States should be proactive to forego some taxes on account of interstate transport of commodities at least for the time being till the prices of inputs used in agriculture.